Registered Nurse RN

Registered Nurse, Free Care Plans, Free NCLEX Review, Nurse Salary, and much more. Join the nursing revolution.

Next Generation NCLEX Case Study Sample Questions

One of the big changes on the Next Generation NCLEX exam is a shift toward case studies. Case studies often require a deeper level of critical thinking, and understanding diseases on a more in-depth level (especially the pathophysiology) will make these types of questions easier to answer.

In this article, you’ll be able to watch a free video to help you prepare for the new Next Generation NCLEX case study format. Nurse Sarah will walk you step-by-step through each scenario and help you understand how to use critical thinking and nursing knowledge to answer these types of questions.

Next Generation NCLEX Case Study Review Questions Video

NGN Case Study Sample Questions and Answers

First, let’s take a look at our case study summary below:

Case Study Summary:

A 68-year-old male is admitted with shortness of breath. He reports difficulty breathing with activity, lying down, or while sleeping. He states that in order to “breathe easier,” he has had to sleep in a recliner for the past week. The patient has a history of hypertension, myocardial infarction (2 years ago), and cholecystectomy (10 years ago). The patient is being transferred to a cardiac progressive care unit for further evaluation and treatment.

Question 1 of 6: The nurse receives the patient admitted with shortness of breath. What findings are significant and require follow-up? The options are listed below. Select all that apply.

To answer this first question in the NGN case study, let’s look at the information provided in the nursing notes and vital signs tabs provided:

next generation nclex, ngn case study, next generation nclex case study, next generation nclex questions and answers, ngn practice

This question is asking us to identify findings that are significant and require the nurse to follow-up. In other words, what is presenting that we can’t ignore but need to investigate further.

Therefore, let’s comb through the nursing notes and vital signs to see what is abnormal and requires follow-up.

First, the patient arrived to the room via stretcher. That’s fine and doesn’t necessarily require follow-up.

Next, the patient is alert and oriented x 4 (person, place, time, event). This tells us that the patient’s neuro status is intact so far. Therefore, the shortness of breath isn’t affecting the patient’s mental function yet (we have enough oxygen on board right now for brain activity).

However, the nurse has noticed the shortness of breath with activity and talking, which should not normally happen. This tells us something is wrong and is significant enough to require follow-up. We want to know why is this happening, is it going to get worse, etc.

The patient’s weight and vital signs were collected (this is good). Weight is 155 lbs. and BMI is within a healthy range (doesn’t tell us too much but may be useful later). The patient is also connected to a bedside monitor, so they need to be monitored constantly like on a progressive care unit.

The monitor shows sinus tachycardia . This is significant because it seems the patient’s shortness of breath is causing the heart to compensate by increasing the heart rate to provide more oxygen (hence the lungs may be compromised).

Then we find out that the lungs are indeed compromised because crackles are heard in both lungs , and this may be why our patient is short of breath. This is significant (could the patient have pulmonary edema?)

Then we find out the nurse has noted an S3. This is an extra heart sound noted after S2. And what jumps out to me about this is that it is usually associated with volume overload in the heart like in cases of heart failure . However, S3 may be normal in some people under 40 or during pregnancy, but that’s not the case with our patient based on what we read in the case summary.

Therefore, based on everything I’m reading in this case study, I’m thinking this patient may have heart failure, but we need those test results back (especially the echo and chest x-ray, and hopefully a BNP will be in there too).

We are also told that the patient has an 18 gauge IV inserted (which is good thing to have so we can give medications if required), orders have been received, labs drawn, and testing results are pending.

next generation nclex, nclex prep, nclex case study questions, nclex questions and answers, ngn review,

Now let’s look at the “Vital Signs” tab above, and ask yourself what is normal vs. abnormal for this patient (adult male).

  • The heart rate is high at 112 (tachycardia), and should normally be 60-100 bpm (see heart rhythms ).
  • Blood pressure is higher than normal (normal is 120/80), which indicates hypertension.
  • Oxygen saturation is 94% (this is on the low side as we’d normally want around 95% or higher, and the patient is on 4 L nasal cannula, which tells us the lungs are not okay).
  • Respiratory rate is increased (26 breaths per minute)…normal is 12-20 breaths per minute.

Based on the information we were provided, I’ve selected the answers below. These findings are significant and definitely require follow-up by the nurse.

next generation nclex questions and answers, next generation nclex answers, next generation nclex sample questions, ngn questions

When answering these NGN case study questions, it’s helpful to think of the ABCDE (airway, breathing, circulation, etc.) as all of these fall into that category. If we don’t follow-up on the shortness of breath, crackles, respiratory rate, o2 saturation (94% on 4 L nasal cannula), the respiratory system can further decline.

In addition, the sinus tachycardia, S3 gallop, and hypertension could indicate fluid overload in the heart. This may cause the heart to tire out and lead the lethal rhythm. On the other hand, temperature, pain, weight, and BMI are not abnormal and do not require follow-up.

See the Complete Next Generation NCLEX Case Study Review

Each question in the case study builds on the previous question. To see how these questions evolve based on the patient’s condition and labs, watch the entire Next Generation NCLEX Case Study Review video on our YouTube Channel (RegisteredNurseRN).

NCLEX Practice Quizzes

We’ve developed many free NCLEX review quizzes to test your knowledge on nursing topics and to help you prepare for the Next Generation NCLEX exam.

Nurse Sarah’s Notes and Merch

fluid electrolytes nursing nclex, notes, mnemonics, quizzes, nurse sarah, registerednursern

Just released is “ Fluid and Electrolytes Notes, Mnemonics, and Quizzes by Nurse Sarah “. These notes contain 84 pages of Nurse Sarah’s illustrated, fun notes with mnemonics, worksheets, and 130 test questions with rationales.

You can get an eBook version here or a physical copy of the book here.

Please Share:

  • Click to print (Opens in new window)
  • Click to share on Facebook (Opens in new window)
  • Click to share on Twitter (Opens in new window)
  • Click to share on Pinterest (Opens in new window)
  • Click to share on Reddit (Opens in new window)
  • Click to share on LinkedIn (Opens in new window)
  • Click to share on WhatsApp (Opens in new window)
  • Click to share on Pocket (Opens in new window)
  • Click to share on Telegram (Opens in new window)

Disclosure and Privacy Policy

Important links, follow us on social media.

  • Facebook Nursing
  • Instagram Nursing
  • TikTok Nurse
  • Twitter Nursing
  • YouTube Nursing

Copyright Notice

  • Skip to content

MNWC

State Resources

National Resources

Nursing Organizations

  • MNWC Initiatives

Maryland Nursing Workforce Center

  • NextGen NCLEX

NextGen NCLEX Test Bank

The purpose of this project was to develop a repository of nextgen nclex questions that can be accessed by all faculty and students in maryland..

The questions can be used by faculty to prepare students to understand the new format of Next Generation (NextGen) test items that are like those that will be used by the National Council of State Boards of Nursing (NCSBN) licensing exam beginning in April 2023 to test students’ ability to make clinical judgments.

Disclaimer: The items in the test bank are accessible to all through this nonsecure website. The test questions are not recommended to be used for summative assessments.

The test bank is composed of case studies with six questions each that follow the NCSBN Clinical Judgment Measurement Model steps:

  • recognize cues
  • analyze cues
  • prioritize hypotheses
  • generate solutions
  • take action
  • evaluate outcomes.

In addition, seven questions for reviewing bow-tie or trend items are included. All case studies were subjected to rigorous review both by the project team and subject matter experts.

The names of the case studies are provided with hyperlinks to all items.

  • Using the Maryland NextGen Test Bank
  • Student Instructions for Using the Test Bank

NCLEX Next Gen Test Bank

NCLEX Practice Questions Test Bank for Free

NCLEX-RN_

Welcome to our collection of free NCLEX practice questions to help you achieve success on your NCLEX-RN exam! This updated guide for 2024 includes 1,000+ practice questions, a primer on the NCLEX-RN exam, frequently asked questions about the NCLEX, question types, the NCLEX-RN test plan, and test-taking tips and strategies.

Table of Contents

Nclex-rn practice questions test bank, what is nclex.

  • What is Next Generation NCLEX (NGN)? 

Safe and Effective Care Environment

  • Health Promotion and Maintenance 
  • Psychosocial Integrity 

Physiological Integrity

Clinical judgment, integrated processes, item writers for nclex, computer adaptive test (cat), 95% confidence interval rule, maximum-length exam, run-out-of-time (r.o.o.t.) rule, zero/one (0/1) scoring, plus/minus (+/-) scoring, rationale scoring, pretest items, testing time, unfolding case study, stand-alone items, multiple response select all that apply.

  • Multiple Response Select N 

Multiple Response Grouping

  • Highlight in Text 

Highlight in Table

Matrix multiple response, matrix multiple choice.

  • Drag and Drop Cloze 

Drag and Drop Rationale

Drop down cloze, drop down rationale, drop down in table, chart or exhibit questions, graphic option, select all that apply or multiple-response, fill-in-the-blank, ordered-response, how to register for the nclex, nclex-rn resources, want more practice questions, recommended resources.

Looking for the complete collection of practice questions? For more NCLEX practice questions, please visit our Nursing Test Bank here .

We have included more than 1,000+ NCLEX practice questions covering different nursing topics for this nursing test bank ! We’ve made a significant effort to provide you with the most challenging questions along with insightful rationales for each question to reinforce learning .

We recommend you do all practice questions before you take the actual exam. Doing so will help reduce your test anxiety and help identify nursing topics you need to review. To make the most of the practice exams, try to minimize mistakes to less than 15 questions and take your time answering the questions, especially when reading the rationales.

Quiz Guidelines

Before you start, here are some examination guidelines and reminders you must read:

  • Practice Exams : Engage with our Practice Exams to hone your skills in a supportive, low-pressure environment. These exams provide immediate feedback and explanations, helping you grasp core concepts, identify improvement areas, and build confidence in your knowledge and abilities.
  • You’re given 2 minutes per item.
  • For Challenge Exams, click on the “Start Quiz” button to start the quiz.
  • Complete the quiz : Ensure that you answer the entire quiz. Only after you’ve answered every item will the score and rationales be shown.
  • Learn from the rationales : After each quiz, click on the “View Questions” button to understand the explanation for each answer.
  • Free access : Guess what? Our test banks are 100% FREE. Skip the hassle – no sign-ups or registrations here. A sincere promise from Nurseslabs: we have not and won’t ever request your credit card details or personal info for our practice questions. We’re dedicated to keeping this service accessible and cost-free, especially for our amazing students and nurses. So, take the leap and elevate your career hassle-free!
  • Share your thoughts : We’d love your feedback, scores, and questions! Please share them in the comments below.

Included NCLEX-RN question sets for this nursing test bank are as follows. Please click on the links below and you’ll be redirected to the quiz page:

NEW UPDATE: Introducing Challenge Exams and Practice Exams

Engage with these new modes to enhance your learning journey, each tailored to meet different needs and propel you towards academic achievement.

  • Dive into Practice Exam Mode for a supportive learning experience. Here, real-time feedback aids in understanding core concepts and building confidence. It’s your space for growth and preparation.
  • Step up to Challenge Exam Mode to test your mastery. With rigorous questions, a leaderboard, and a timer, it’s designed to assess your readiness and foster a competitive spirit.

Test your mastery with these Challenge Exams :

Build your confidence and grasp core concepts with these Practice Exams :

The National Council Licensing Examination (NCLEX) is a comprehensive test administered by the National Council of State Boards of Nursing (NCSBN). It assesses whether candidates possess the necessary knowledge and skills to provide safe and effective nursing care at the entry level. The NCLEX is available in two versions: the NCLEX-RN for registered nurses and the NCLEX-PN for practical/vocational nurses.

The NCSBN, composed of nursing regulatory bodies from all 50 states in the US, the District of Columbia, and four US territories, is responsible for safeguarding the public by ensuring safe nursing care. It sets the standards and guidelines for nursing licensure and develops the NCLEX examinations.

Becoming a registered nurse (RN) requires meeting specific licensure requirements determined by the licensing authorities in each jurisdiction governed by the NCSBN. One of these requirements is passing the NCLEX-RN, which evaluates the competencies necessary for practicing safely and effectively as an entry-level RN. The NCLEX-RN is used by member boards of nursing and many Canadian nursing regulatory bodies to inform their licensure decisions.

What is Next Generation NCLEX (NGN)?

The Next Generation NCLEX (NGN) is currently in effect this April 2023 for RN and LPN /LVN candidates. The change in the NCLEX is driven by the need to adapt to the increasing complexity of client care, advancements in healthcare practice, and the demand for safe clinical decision-making. The NGN aims to address the declining ability of new nursing graduates to make safe clinical decisions by integrating clinical judgment as a key competency. The NGN test format will remain adaptive but with fewer test items. Candidates will encounter Unfolding Case Studies and Stand-Alone Items , scored using partial credit with three different scoring rules. These changes in the NGN aim to assess critical thinking and the ability to make safe clinical judgments during various phases of client care.

In summary, the following are the key changes between the previous NCLEX format and the Next Generation NCLEX (NGN): 

  • Same adaptive test format but reduced number of items. The Next Generation NCLEX (NGN) will retain the adaptive test format, similar to the current NCLEX, but with fewer test items. Candidates will answer between 85 and 150 items, with 15 as pretest or unscored items. 
  • Case studies. All candidates will encounter three (3) Unfolding Case Studies, each consisting of six test items (a total of 18 items). Clinical situation information will be presented to candidates in a medical record format resembling a table-like structure. These situations are designed to assess their ability to think critically and make safe clinical decisions across different phases of client care. In addition to the case studies, some candidates may have Stand-Alone Items for their NGN exam.
  • New NGN item types. The NGN exam will feature new item types, which include questions in unfolding cases and stand-alone items, highlight, cloze, matrix, bow-tie, drag and drop, and extended multiple responses. 
  • Scoring differences. NGN test items will be scored differently compared to the current NCLEX. They will utilize partial credit scoring with three different scoring rules: the 0/1 scoring rule, the +/- scoring rule, and the rationale scoring rule. These are explained further below. 

NCLEX-RN Test Plan

The NCLEX test plan is a content guideline to determine the distribution of test questions. NCSBN uses the “Client Needs” categories to ensure the NCLEX covers a full spectrum of nursing activities. It is a summary of the content and scope of the NCLEX to serve as a guide for candidates preparing for the exam and to direct item writers in the development of items.  Additionally, to assess the knowledge, skills, abilities, and clinical judgment necessary for entry-level nursing practice, the NCLEX-RN Test Plan utilizes Bloom’s taxonomy for the cognitive domain. This taxonomy provides a foundation for writing and coding examination items, focusing on application-level or higher-level cognitive abilities that require advanced thought processing.

The content of the NCLEX-RN is organized into four major Client Needs categories: Safe and Effective Care Environment, Health Promotion and Maintenance, Psychosocial Integrity, and Physiological Integrity. Some of these categories are divided further into subcategories. Below is the NCLEX-RN test plan effective as of April 2023 until March 2026: 

There are two subcategories under Safe and Effective Care Environment.

  • Management of Care category includes content that tests the nurse’s knowledge and ability to direct nursing care that enhances the care delivery setting to protect clients, significant others, and healthcare personnel. 
  • Safety and Infection Control category includes content that tests the nurse’s ability required to protect clients, families, and healthcare personnel from health and environmental hazards. 

Health Promotion and Maintenance

Health Promotion and Maintenance category includes content that tests the nurse’s ability to provide and direct nursing care to the client that incorporates knowledge of expected growth and development, preventing and early detection of health problems, and strategies to achieve optimal health. 

Psychosocial Integrity

The Psychosocial Integrity category is content related to the promotion and support for the emotional, mental, and social well-being of the client experiencing stressful events and clients with acute or chronic mental illness . 

In the Physiological Integrity category are items that test the nurse’s ability to promote physical health and wellness by providing care and comfort, reducing risk potential, and managing health alterations. There are four subcategories under Physiological Integrity. 

  • Basic Care and Comfort are content to test the nurse’s ability to provide comfort and assistance to the client in performing activities of daily living . 
  • Pharmacological and Parenteral Therapies category includes content to test the nurse’s ability to administer medications and parenteral therapies (IV therapy, blood administration, and blood products). 
  • Reduction of Risk Potential category includes content to test the nurse’s ability to prevent complications or health problems related to the client’s condition or prescribed treatments or procedures. 
  • Physiological Adaptation category includes questions that test the nurse’s ability to provide care to clients with acute, chronic, or life-threatening conditions. 

Clinical judgment is a central component of the NCLEX-RN Test Plan, reflecting the evolving demands and complexity of nursing practice. Nurses must engage in an iterative, multi-step process that utilizes nursing knowledge to observe and assess situations, identify client concerns, and generate evidence-based solutions to ensure safe client care. Clinical judgment questions are represented through Unfolding Case Studies or individual Stand-Alone Items , with case studies addressing each step of clinical judgment. The following are the six clinical judgment cognitive skills: 

  • Recognize cues “What matters most?” Identifying relevant and important information from various sources, such as medical history, laboratory studies, and vital signs.
  • Analyze cues “What could it mean?” Organizing and connecting recognized cues to the client’s clinical presentation.
  • Prioritize hypotheses “Where do I start?”   Evaluating and prioritizing hypotheses based on urgency, likelihood, risk, difficulty, time constraints, and other factors.
  • Generate solutions “What can I do?” Identifying expected outcomes and using hypotheses to define a set of interventions aligned with the expected outcomes.
  • Take action “What will I do?” Implementing the highest-priority solution(s) identified.
  • Evaluate outcomes “Did it help?” Comparing observed outcomes to expected outcomes to assess the effectiveness of interventions.

Additionally, In contrast to the nursing process or ADPIE , the NCSBN focuses on AAPIE ( assessment , analysis, planning, intervention, evaluation), wherein nursing diagnosis is not tested at the current NGN because it is not considered a universal language used in health care or the nursing profession. In nursing practice and for the NGN, students must use their pathophysiology knowledge to analyze client assessments and connect them with common conditions in healthcare settings. 

Integral to nursing practice, several processes are integrated throughout the Client Needs categories and subcategories:

  • Caring. In an atmosphere of mutual respect and trust, nurses interact with clients, providing encouragement, hope, support, and compassion to help achieve desired outcomes.
  • Clinical Judgment . An observed outcome of critical thinking and decision-making, clinical judgment (discussed above) is a dynamic and iterative process. It involves using nursing knowledge to observe and assess situations, prioritize client concerns, generate evidence-based solutions, and deliver safe client care.
  • Communication and Documentation. Verbal and nonverbal interactions between nurses, clients, significant others, and the healthcare team. Accurate and comprehensive documentation ensures adherence to practice standards and accountability in care provision.
  • Culture and Spirituality. Recognizing and considering the unique preferences, standards of care, and legal considerations of clients (individuals, families, groups, and populations) in the context of their culture and spirituality.
  • Nursing Process. A systematic approach to client care encompassing assessment, analysis, planning, implementation, and evaluation.
  • Teaching/Learning. Facilitating acquiring knowledge, skills, and abilities that promote behavioral change.

Who writes questions for the NCLEX? The NCSBN sets the criteria and selection process for item writers who are registered nurses. Many of them are nursing educators with an advanced degrees in nursing, so if you’ve completed an accredited nursing program, you have already taken several tests written by nurses with backgrounds similar to those who write for the NCLEX.

The NCLEX-RN is a crucial step for registered nurse (RN) candidates seeking licensure. Administered through computerized adaptive testing ( CAT ), this examination utilizes computer technology and measurement theory to create a personalized and accurate assessment of each candidate’s knowledge and skills. 

The CAT system employed in the NCLEX-RN ensures that each candidate receives a unique examination tailored to their abilities. As the test progresses, the computer selects items from a large item pool based on the candidate’s previous answers. These items are classified according to the test plan categories, difficulty levels, and clinical judgment steps. The CAT system continually recalculates the candidate’s ability estimate and selects subsequent items accordingly, creating an exam that aligns with the NCLEX-RN Test Plan requirements. This dynamic approach allows candidates to demonstrate their competence effectively.

Additionally, unlike traditional fixed-length exams, which assign the same items to all candidates, CAT selects items based on the candidate’s ability, resulting in a more accurate assessment. The CAT scoring algorithm estimates the candidate’s ability by considering all previous answers and the difficulty level of those items. By administering items that challenge the candidate appropriately, the exam gathers maximum information about their ability.

Pass or Fail Decisions: How to Pass the NCLEX?

Passing the NCLEX-RN requires you, the candidate, to meet a specific passing standard established by the NCSBN Board of Directors (BOD). The passing standard represents the minimum ability level necessary for safe and effective entry-level nursing practice. The BOD reevaluates this standard every three years, considering various factors such as a standard-setting exercise conducted by experts and psychometricians, historical data on candidate performance, and information on the educational readiness of aspiring nurses. Once the passing standard is set, it is uniformly applied to all candidates during the scoring process. The NCSBN indicates that these three rules govern pass-or-fail decisions: the 95% Confidence Interval Rule , Maximum-Length Exam Rule , and Run-Out-Of-Time Rule .

In this scenario, the computer stops administering test questions when it is 95% certain that your ability is clearly above the passing standard or clearly below the passing standard. 

When your ability is close to the passing standard, the CAT gives you items until the maximum number of items is reached. At this point, the computer disregards the 95% confidence rule and decides whether you pass or fail by your final ability estimate. If your final ability estimate is above the passing standard, you pass; if it is below, you fail. 

In the event that a candidate runs out of time before completing the maximum number of items and the computer has not reached a 95% certainty regarding the candidate’s pass or fail status, alternative criteria come into play.

  • If the candidate has not answered the minimum required number of items, the candidate will automatically fail.
  • If the candidate has answered at least the minimum required number of items, the final ability estimate will be determined based on all the responses given before the exam time expires. If the score meets or exceeds the passing standard, the candidate will pass. Otherwise, the candidate will fail.

New Scoring System in the Next Generation NCLEX

The Next Generation NCLEX (NGN) incorporates different item formats, and for items that have multiple correct answers, partial credit scoring is implemented . Unlike the previous exam format, where items are scored as either all correct or all incorrect, the NGN will use polytomous scoring models to evaluate partial understanding. For example, in the previous version of NCLEX (multiple response) SATA (select-all-that-apply) items, only endorsing all the correct options results in a correct score. However, the new approach will acknowledge partial understanding, allowing differentiation between candidates with different numbers of correct options. This change will provide a more accurate assessment of candidates’ knowledge, skills, and abilities.

Three methods are used to assign partial credit for these items: plus/minus scoring, zero/one scoring, and rationale scoring. Understanding the scoring models is crucial for NCLEX-RN takers to know how their answers are evaluated and how scores are determined.

The 0/1 scoring rule is used for multiple-choice items. Choosing the correct option earns one point while selecting an incorrect option gives a score of zero. This rule applies to single response items, including the example multiple-choice item.

The +/– scoring rule measures a candidate’s ability to identify relevant information. It assigns points for correct options and deducts points for incorrect options. It is used for multi-point items, and the total score is calculated by adding correct options and subtracting incorrect ones. The lowest possible score is zero. This rule is applied to NGN question types such as multiple-response and select all that apply.

The rationale scoring rule is used for questions that assess paired information. To earn a score of one point, both answer options must be correct. If either option is incorrect, the score is 0.

How Many Questions Are on the NCLEX?

All RN candidates must answer a minimum of 85 items , while a maximum of 150 items be administered during the five-hour time allotment. Each NCLEX-RN exam includes 15 pretest items (unscored), which are indistinguishable from operational (scored) items.

To ensure the effectiveness of computerized adaptive testing (CAT) in the NCLEX, the difficulty level of each item needs to be determined beforehand. This is achieved by administering these items as pretest items to a large group of NCLEX candidates. Since the difficulty of pretest items is initially unknown, they are not part of your score and are not considered when evaluating a candidate’s ability or determining pass-fail outcomes. Once a sufficient number of responses are collected, the pretest items undergo statistical analysis and calibration. If they meet the required NCLEX statistical standards, they can be used as operational items in future exams. Therefore, candidates must approach every item with their best effort, regardless of whether it is a pretest or an operational item.

The length of the NCLEX-RN examination varies for each candidate based on their responses, but there is a five-hour time limit for the exam, including all breaks. To ensure completion within the allotted time, candidates must maintain a reasonable pace, spending approximately one to two minutes per item . It’s important to note that the length of the examination does not determine pass or fail outcomes; candidates can succeed or fail regardless of the examination’s duration.

Case Studies in NGN

The Next Generation NCLEX (NGN) has introduced case studies that present practical clinical scenarios and include test items aligned with six cognitive abilities of the clinical judgment model. The scenario can occur in various settings, and the client’s outcome can vary, including improvement, stability, or decline with complications. Laboratory results, if included, are presented in a table, and abnormal values are highlighted with an “L” or “H”. There are two types of case studies: Unfolding Case Study with six test items assessing specific skills and Stand-Alone Items with a single test item independent of a case study.

An Unfolding Case Study presents a realistic client situation with evolving data resembling a medical record. It includes multiple phases over time, reflecting changes in the client’s condition. The case study evaluates all six cognitive skills of the clinical judgment model through six questions. Each test-taker receives three (3) unfolding case studies, totaling 18 items across those cases. 

Stand-Alone Items are case studies with a single question, with the item presented as a bow-tie or a trend question type. Candidates who take more than the minimum number of items on the NCLEX receive around six to seven stand-alone cases. 

Question Types in the NCLEX-RN

The different question types for the NCLEX-RN include the following: 

Extended Multiple Response

In Extended Multiple Response item types, test-takers can choose one or more options in Multiple Response Select All That Apply, select a specific number of items in Multiple Response Select N, or choose options from different groupings presented in a table in Multiple Response Grouping, with scoring rules based on the selected type. Extended multiple response may include the following formats:

The Multiple Response Select All That Apply item type allows test-takers to choose one or more answer options. It uses the +/- Scoring Rule, where selecting correct information earns points (+1), and selecting incorrect information results in points deducted (-1). The maximum points achievable are equal to the number of correct options, and the minimum score is 0, with no negative scores. The item must have a minimum of five options and can have up to ten options, with the possibility of all ten options being correct.

Multiple Response Select N

The Multiple Response Select N item type informs the test-taker about the specific number of items (N) that can be selected. The question provides a range of options, with a minimum of five and a maximum of ten options. It uses the 0/1 Scoring Rule, where each correct response earns 1 point, while any incorrect response results in 0 points.

The Multiple Response Grouping utilizes a table to present options, with a minimum of two and a maximum of five groupings. Each grouping consists of a minimum of two and a maximum of four options, ensuring an equal number of options within each grouping. Test-takers are required to select at least one option from each grouping. The item follows the +/- Scoring Rule, meaning there is no negative score per grouping. The total score for the item is determined by summing the points earned within each grouping. The maximum score achievable equals the number of keys (N).

For this NGN item type, the test-taker is presented with information and must highlight specific parts of the information provided.  Highlight question type include the following formats:

Highlight in Text

It involves a paragraph of information where the test-taker must select (highlight) specific parts of the text based on the question’s requirements. The item can include a maximum of ten options for selection and uses the +/- Scoring Rule. 

The Highlight in Table item type involves a table of information. The candidate is tasked with selecting (highlighting) specific parts of the text within the table based on the question’s prompt. The item can include a maximum of ten options for selection. The table has two columns, including a header, and can have up to five rows. This item type uses the +/- Scoring Rule. 

Matrix/Grid

In the Matrix NGN item type, test-takers are presented with a grid-like structure to which they must respond. There are two variations: Matrix Multiple Response, where each column can have multiple correct responses, and Matrix Multiple Choice, where test-takers select one answer option per row. Extended multiple response may include the following formats:

The Matrix Multiple Response item type consists of response columns, where each column can have multiple correct responses. The item can have between two and ten columns and four to seven rows. In each column, at least one response option must be selected, but it is possible to select one or more responses per column. The item follows the +/- Scoring Rule, with no negative score per column. The total score for the item is determined by summing the points earned within each column. The maximum score achievable is equal to the number of keys (N).

The item includes a minimum of four rows and a maximum of ten rows. It can have either two or three options/columns. Test-takers are allowed to select one answer option per row. The item follows the 0/1 scoring rule, where the test-taker earns 1 point for each correct response and 0 points for incorrect responses. The total score for the item is obtained by summing the scores over the rows. The maximum score achievable is equal to the number of rows.

Drag and Drop

In the Drag and Drop NGN question type, test-takers are presented with options to drag and drop into specific targets. There are two variations: Drag and Drop Cloze, where options are placed in response targets to complete sentences and Drag and Drop Rationale, where options are matched to causes and effects. Drag and drop may include the following formats:

Drag and Drop Cloze

The Drag and Drop Cloze item type includes a range of four to ten options. It can have one or more response targets where the options are dragged to. The item must have a minimum of one sentence with one target per sentence and up to five sentences, each with one target per sentence. The item follows the 0/1 Scoring Rule, where a correct response earns 1 point and incorrect responses earn 0 points. The total score is obtained by summing the scores across all targets, with the maximum score achievable being the number of targets.

The Drag and Drop Rationale type involves one sentence with one cause and one effect or one sentence with one cause and two effects. The sentence can be a single dyad (one sentence with two targets) or a single triad (one sentence with three targets). Each drag & drop target can have three to five options. The item follows the Rationale Scoring Rule where both answer options must be correct to earn a point. 

The Drop Down question type in NGN includes three variations: Drop Down Cloze, Drop Down Rationale, and Drop Down in Table. Drop down may include the following formats:

The Drop Down Cloze item type presents one or two sentences of information that require completion using drop-down options. Each drop-down can contain three to five options. The item must have a minimum of one sentence with one drop-down per sentence and can include up to five sentences, each with one drop-down. The item follows the 0/1 Scoring Rule, where the score is calculated by summing the correct responses across all drop-downs. The maximum score achievable is equal to the number of drop-downs.

The Drop Down Rationale item type presents either one sentence with one cause and one effect or one sentence with one cause and two effects. Each drop-down can contain three to five options. The scoring follows the Rationale Scoring Rule, where X and Y must be correct to earn one point. The maximum points that can be earned are 1 point for a dyad (one sentence with two targets) and 2 points for a triad (one sentence with three targets).

The Drop Down in Table item type presents a table of information with drop-down options located in various parts of the table. The item must have a minimum of three columns and three rows and a maximum of five columns and four rows. One column serves as the header column. Each row contains one drop-down. The item follows the 0/1 Scoring Rule, where the score is calculated by summing the correct responses across all drop-downs. The maximum score achievable is equal to the number of drop-downs.

The Bow-tie item type is visually designed to resemble a bow-tie shape. It consists of five options on the left side, five options on the right side, and four options in the middle well of the bow-tie. The item addresses all of the cognitive skills within a single item. The Bow-tie item follows the 0/1 scoring rule. The left and right wells have two answer options each, while the middle well has one answer option, resulting in a maximum possible score of 5 points. Each correct response earns 1 point, while incorrect responses earn 0 points.

The Trend item type presents information in a medical record or flow sheet format that spans over time, requiring the test-taker to analyze information across different points in time. The item addresses multiple cognitive skills within a single item. It can utilize any NGN item type except for the bow-tie. The scoring rule for the Trend item depends on the specific item type used.

Multiple-Choice Questions

Many questions on the NCLEX are in multiple-choice format. This traditional text-based question will provide you data about the client’s situation, and you can only select one correct answer from the given four options. Multiple-choice questions may vary and include: audio clips, graphics, exhibits, or charts.

Multiple-choice question format for the NCLEX

A chart or exhibit is presented along with a problem. You’ll be provided with three tabs or buttons that you need to click to obtain the information needed to answer the question. Select the correct choice among four multiple-choice answer options. 

Chart or Exhibit Alternate Format for the NCLEX

In this format, four multiple-choice answer options are pictures rather than text. Each option is preceded by a circle that you need to click to represent your answer.

In an audio question format, you’ll be required to listen to a sound to answer the question. You’ll need to use the headset provided and click on the sound icon for it to play. You’ll be able to listen to the sound as many times as necessary. Choose the correct choice from among four multiple-choice answer options. 

For the video question format, you must view an animation or a video clip to answer the following question. Select the correct choice among four multiple-choice answer options. 

Multiple-response or select all that apply (SATA) alternate format question requires you to choose all correct answer options that relate to the information asked by the question. There are usually more than four possible answer options. No partial credit is given in scoring these items (i.e., selecting only 3 out of the 5 correct choices), so you must select all correct answers for the item to be counted as correct. 

Select All That Apply Question format for the NCLEX

Tips when answering Select All That Apply Questions

  • You’ll know it’s a multiple-response or SATA question because you’ll explicitly be instructed to “Select all that apply.”
  • Treat each answer choice as a True or False by rewording the question and proceed to answer each option by responding with a “yes” or “no”. Go down the list of answer options one by one and ask yourself if it’s a correct answer.
  • Consider each choice as a possible answer separate to other choices. Never group or assume they are linked together.

The fill-in-the-blank question format is usually used for medication calculation, IV flow rate calculation, or determining the intake-output of a client. You’ll be asked to perform a calculation in this question format and type in your answer in the blank space provided. 

Fill-in-the-Blank question format for the NCLEX

Tips when answering Fill-in-the-Blank

  • Always follow the specific directions as noted on the screen. 
  • There will be an on-screen calculator on the computer for you to use. 
  • Do not put any words, units of measurements, commas, or spaces with your answer, type only the number. Only the number goes into the box.
  • Rounding an answer should be done at the end of the calculation or as what the question specified, and if necessary, type in the decimal point. 

In an ordered-response question format, you’ll be asked to use the computer mouse to drag and drop your nursing actions in order or priority. Based on the information presented, determine what you’ll do first, second, third, and so forth. Directions are provided with the question. 

Ordered-response question format for the NCLEX

Tips when answering Ordered-Response questions

  • Questions are usually about nursing procedures. Imagine yourself performing the procedure to help you answer these questions.
  • You’ll have to place the options in correct order by clicking an option and dragging it on the box on the right. You can rearrange them before you hit submit for your final answer.

A picture or graphic will be presented along with a question. This could contain a chart, a table, or an illustration where you’ll be asked to point or click on a specific area. Figures may also appear along with a multiple-choice question. Be as precise as possible when marking the location. 

Hotspot Alternate Question Format for the NCLEX

Tips when answering Hotspot questions

  • Mostly used to evaluate your knowledge of anatomy, physiology, and pathophysiology.
  • Locate anatomical landmarks to help you select the location needed by the item.

So you’ve finally decided to take the NCLEX, the next step is registration or application for the exam. The following are the steps on how to register for the NCLEX, including some tips:

  • Application to the Nursing Regulatory Board (NRB). The initial step in the registration process is to submit your application to the state board of nursing in the state in which you intend to obtain licensure. Inquire with your board of nursing regarding the specific registration process as requirements may vary from state to state.
  • Registration with Pearson VUE. Once you have received the confirmation from the board of nursing that you have met all of their state requirements, proceed, register, and pay the fee to take the NCLEX with Pearson VUE . Follow the registration instructions and complete the forms precisely and accurately.
  • Authorization to Test. If you were made eligible by the licensure board, you will receive an Authorization to Test (ATT) form from Pearson VUE. You must test within the validity dates (an average of 90 days) on the ATT. There are no extensions or you’ll have to register and pay the fee again. Your ATT contains critical information like your test authorization number, validity date, and candidate identification number.
  • Schedule your Exam Appointment. The next step is to schedule a testing date, time, and location at Pearson VUE. The NCLEX will take place at a testing center , you can make an exam appointment online or by telephone. You will receive a confirmation via email of your appointment with the date and time you choose including the directions to the testing center. * Changing Your Exam Appointment. You can change your appointment to test via Pearson VUE or by calling the candidate services. Rules for scheduling, rescheduling, and unscheduling are explained further here . Failing to arrive for the examination or failure to cancel your appointment to test without providing notice will forfeit your examination fee and you’ll have to register and pay again. 
  • On Exam Day. Arrive at the testing center on your exam appointment date at least 30 minutes before the schedule. You must have your ATT and acceptable identification (driver’s license, passport, etc) that is valid, not expired, and contains your photo and signature.
  • Processing Results. You will receive your official results from the board of nursing after six weeks.

Important list and resources you need to know if you’re taking the NCLEX: 

  • 2023 NCLEX Candidate Bulletin. This resource is a comprehensive guide for the NCLEX, providing essential information and contact details for candidates. It covers registration procedures, test fees, scheduling, testing accommodations, exam rules, and what to bring to the test site. The guide also includes details on the check-in process, breaks, technical issues, and the testing environment. It explains the results reporting process and retake-policy and provides information on the content, development, and test plans of the NCLEX. Additionally, it discusses the passing standard, item formats, and decision rules for passing or failing the exam.
  • Sample Questions and Exam Preview. Get a headstart on your exam preparation with these sample test packs by the NCSBN. 
  • Candidate Tutorial. With the new item types, we highly recommend testing how the Next Generation NCLEX (NGN) works. This tutorial guides you on interacting with different question types in your NCLEX exam. It provides a representative sample of items you may encounter during the test, allowing you to practice their functionality.

Please visit our Nursing Test Bank page if you’re looking to answer more practice questions from different topics and different question formats.

Recommended books and resources for your NCLEX success:

Disclosure: Included below are affiliate links from Amazon at no additional cost from you. We may earn a small commission from your purchase. For more information, check out our privacy policy .

Saunders Comprehensive Review for the NCLEX-RN Saunders Comprehensive Review for the NCLEX-RN Examination is often referred to as the best nursing exam review book ever. More than 5,700 practice questions are available in the text. Detailed test-taking strategies are provided for each question, with hints for analyzing and uncovering the correct answer option.

case study next generation nclex sample questions

Strategies for Student Success on the Next Generation NCLEX® (NGN) Test Items Next Generation NCLEX®-style practice questions of all types are illustrated through stand-alone case studies and unfolding case studies. NCSBN Clinical Judgment Measurement Model (NCJMM) is included throughout with case scenarios that integrate the six clinical judgment cognitive skills.

case study next generation nclex sample questions

Saunders Q & A Review for the NCLEX-RN® Examination This edition contains over 6,000 practice questions with each question containing a test-taking strategy and justifications for correct and incorrect answers to enhance review. Questions are organized according to the most recent NCLEX-RN test blueprint Client Needs and Integrated Processes. Questions are written at higher cognitive levels (applying, analyzing, synthesizing, evaluating, and creating) than those on the test itself.

case study next generation nclex sample questions

NCLEX-RN Prep Plus by Kaplan The NCLEX-RN Prep Plus from Kaplan employs expert critical thinking techniques and targeted sample questions. This edition identifies seven types of NGN questions and explains in detail how to approach and answer each type. In addition, it provides 10 critical thinking pathways for analyzing exam questions.

case study next generation nclex sample questions

Illustrated Study Guide for the NCLEX-RN® Exam The 10th edition of the Illustrated Study Guide for the NCLEX-RN Exam, 10th Edition. This study guide gives you a robust, visual, less-intimidating way to remember key facts. 2,500 review questions are now included on the Evolve companion website. 25 additional illustrations and mnemonics make the book more appealing than ever.

case study next generation nclex sample questions

NCLEX RN Examination Prep Flashcards (2023 Edition) NCLEX RN Exam Review FlashCards Study Guide with Practice Test Questions [Full-Color Cards] from Test Prep Books. These flashcards are ready for use, allowing you to begin studying immediately. Each flash card is color-coded for easy subject identification.

case study next generation nclex sample questions

178 thoughts on “NCLEX Practice Questions Test Bank for Free”

How much are you charging for it?

Thanks for being generous

Thank you for your kindness & generosity. This is a big help for us, especially those with financial constraints.

How much for this

Thanks Matt Vera for your utmost effort in helping nurses to pass the exam

You’re most welcome! Good luck on your exams!

Hello Matt do you still have the Test Bank questions for 2021. If you do can I have them please and thank you.

Hi Winnie, Please visit our

Thanks heaps! Matt..

The site is very useful.

Thank you mam/sir

This is very helpful site. god bless🤗

Great website wish l knew about it before taking my nclex

Thank you so much for this free site. I found it just as helpful as Kaplan and Sanders which I paid for. I recommended this nurseslabs to some of my nurse friends preparing for the NCLEX. I used it to study and passed my NCLEX October 28. I will continue to use this as a reference. Once again Thank-You. A.Haynes RN

Hey Matt, I am impressed by your generosity. I couldn’t believe these were all free. In all fairness, there should be a tab for donations for the constant upkeep of the web. A very minimal way to show our gratitude.

Although I just started nursing school the i formation is mind blowing and very useful for me already. Thank you for your generosity. I will be using all the practice quizzes and test to prepare myself. Thank you again.

Thank you for this free nclex practice tests! May you have more blessings and success! I will take my exam next month.

Very informative and helpful

Thank you!You’re a big blessing to those who will take the exam.

I can’t thank you enough for this gift you have given us to use . God bless you.

Thank you so much Matt! This is helping me work toward my NCLEX with less pressure.

Thanks a lot can I get reading materials on PDF please

Thanks Matt Vera for ingenious effort.

You are heaven sent, Thank you Matt for helping us.

you guys all thank you so much for this massively hard work and huge success for all the nurses around the world .

Your efforts are immeasurable, you are not only helping nurses to pass their NCLEX exams but also increasing the number of qualified/competent nurses providing healthcare around the globe. We are grateful. I used this site to prepare for internal exams whilst I was in school and it really helped me. I’ll be writing NCLEX soon..

Planning to start this process, I think this will be so helpful to me. Thankyou very much

Thank you so much Nurse matt vera for this qbank,most beneficial, God bless you., if I may ask for some videos or pictures associated with the questions, will also help e.g lung sounds,abdominal sound etc thank you

I loved your blog and thanks for publishing this about NCLEX practice questions!! I am really happy to come across this exceptionally well written content. Thanks for sharing and look for more in future!! Keep doing this inspirational work and share with us.

My name is Daniel m Johnson from Tanzania I’m holding diploma in nursing and midwifery I’m looking for different nurse from different country so as we can share ideas and experience since I’m student I need to no much thing about nursing ….any one who is touched to please help me I wish to be came a good nurse and help different people from different society

What a wonderful thing to do for people! Thank you very much.

This very helpful to me, thank you.

You did a great job Matt…

This is so lovely

Very useful site. Thank you so much.

This is an amazing job Matt!! REALLY INCREDIBLE!!!

With all this I will certainly pass my NCLEX exams.

THANK YOU so so much Matt, this is great beyond measure.. I have been hook to this website. God will bless you and yours abundantly

You’re most welcome! :)

i just want to confirm for #39 it state pt need additional teaching so E should also be included to the answer cause it was contraindicated to MS right? . thank you :)

Yes, this has been corrected. Thank you! :)

Very informative and helpful. Thank you so much Sir/ madam

Thank you for providing regarding NCLEX preparation

Very informative. Thank you for update.

This is for sure the best nurse’s website ever. It really helped me a lot during my training.

Number 58 of NCLEX test 5 does not have the answers listed.

Hi Rebekah, this is now fixed. Thank you for letting us know!

This is very helpful

It’s very informative. Thanks

Thank you Matt Vera for providing information free of charge

Any advice of how well you can do on these question to build up your confidence?

Thanks a lot for great job. May the Lord Almighty shower His blessing upon you.

Thank you, this site is amazing. God bless you

Thanks alot.I,ve got a question What score ranges would be considered a Pass? I scored 69.33% overall. is that a Pass???

75% and above, from the analysis I got from a test after getting below 75

Thank you Nurse Matt. You deserve all the blessings! 🙏🏻😊

This is very helpful. Thank you Nurse Matt

Question 16 on practice test 9 asked about assessment during L sided CHF, yet the correct answer pertained to right sided HF. Just wanted to let you know…. Thanks for the practice questions! Appreciate all the work put into this.

Question 41 of NCLEX-RN Test 2 has two C choices. Thanks.

Hi, I like your questions, they’re very nice and challenging. Learning a lot from them.

I love this site.. Now all friends are learning serious from here

Thanks you guys are very generous

From Kenya, this site is so helpful,am learning a lot here.Thank you especially for the care plans.

Thank you so much

You Matt and your staff are such a blessing to us who will be taking the NCLEX RN exam. This site is very useful and helpful because it is very comprehensive and easy to understand. God bless and may tribe increase. Thank you so much

The site is very informative. I have learned a lot. Test questions are really amazing which I cannot find with other website. However, I got a low score, thus I need to further study especially on the areas wherein I got a below passing grade. I hope you will allow me. Thank you

thank you so much for creating this site for people like us. This excellent job. May God bless you in Jesus name —Amen

The Practice tests are awesome! The rationales are all well-explained which I love because it gives me more knowledge and insight. Thank you for creating this website free of charge.

Thank you so much. It was really helpful. I passed the exam. God bless real good.

Thank you Matt for the awesome and great website! its beyond compare. I want more practice test questions to hone my knowledge in Nursing practice, however, when I am trying to open one of the practice tests, I cant go through it. Could you please allow me? God bless

Thank you Matt for this practice test. Now I know my strength and weaknesses and will concentrate of the system that needs for further study. Can I practice more in practice test no 2? Please allow me to go trough with it. Kudos to you and your staff.

Hi this is what i got, im so nervous due to my nclex exam its gonna be this coming oct. Its doesnt say pass or not.

Your score 48% Categories Basic Care and Comfort 100% Health Promotion and Maintenance 53.85% Pharmacological and Parenteral Therapies 26.67% Physiological Adaptation 54.55% Psychosocial Integrity 100% Reduction of Risk Potential 36.36% Safety and Infection Control 100%

Thanks so much for this useful information. God bless you.

Thank you so much for all you do

The Practice tests are excellent. The rationales are all well-explained which I love because it gives me more knowledge and insight. Thank you for creating this website free of us. Remain blessed.

Like I am obsess with nurselab. A handy powerful tool, to keep you updated, thick and richly packed. You guys made reading so interesting and magnetic and very easy. Please do not stop. I recommend this to all nurses.

Love this. Thank you for sharing your generosity. I’ve been using this since I was still in college and I continue to use it until now.

Thank you so very much for this

I remain speechless but mostly grateful for your unconditional helpful assistance for this awesome richly throughout profound tool and opportunity given to me to increase my confidence, strengthen my knowledge, and be ready for my upcoming exam. Your questions and rationales are beyond my expectations. You are a God’ sent angel. Thank You. God Bless you. I will be forever grateful to you for making my dream come true! 🙌

This website can help me to review for NCLEX and it is for free! Thank you so much!

Amazing and really awesome job to you Matt and your entire team, may God replenish your source for giving us this for free. I have recommended to all my friends and family. Remain blessed and stay safe.

That’s very kind of you. Thank you so much! Bless you!

Your test questions have greatly helped me pass my NCLEX. I passed my NCLEX today and thank you to Nurseslabs. I came across this website by accident and I do not know how but it was divine intervention. I will definitely recommend Nurseslabs to my friends. The quality of the questions and rationales were very good and you would not think this is all for free the kind of work that came with it.

Kudos Nurseslabs and wishing all the future test takers all the best!

Wow Sam, congrats! What materials did you use other than nurseslabs? I’ve been answering nurseslabs also. My exam is on June. I hope I could also make it.

Thank you for this NCLEX sample test,as tool to determine areas i need to review/focus on for the proper exams.

This website can help me to review for NCLEX Thank you so much!

I just wanna ask why is that there are no more rationale posted after answering the question unlike before. Thanks by the way for the free and great Q & A bank. ❤

Sorry, we’re currently having some performance issues with that setup. Until we can resolve it, we have set each quiz to show results and rationales only after you have finished all the questions. Thank you for your understanding.

Hello, how do I get my score after taking the test to know if I am doing well or not

Hi, after you’re done answering all questions, the final item will direct you to the “Quiz Summary,” from there, click on the “Finish Quiz” button to get your final score.

Very great tool to study for the NCLEX. I passed my exam just by focusing on the questions and rationales provided here.

That’s great! Congratulations!

Thank you so much. It’s very helpful. God bless you…

Thank you for being generous. This is a big help.

I got 55/75.

Thank you so much for making this site.

Very helpful. Thank you and God bless…

Thank you so much, Nurse Matt for your generosity. Helping your fellow nurses and aspiring nurses to work abroad is such a selfless and noble act. God bless you and your team!

Thank you for your kind words, JB! Will pass on this message to the team. :) God bless!

The rationale after the question will really help us a lot specially when you are just using the question via online.

Hi Romel, we have temporarily changed this setting so that results and rationales will show only after you’ve finished the quiz. This is because we’re having some performance issues when using the previous setting. We’ll revert the changes once we’re sure the problem is resolved. Thanks for understanding!

I didn’t get my results after answering the 75 questions. Please why?

Hi Esther, did you click on the “Finish Quiz” button?

Thanks for all these questions where/how can i check my answers to these questions?

After answering the last item of the quiz, you’ll be redirected to the quiz summary. From there, click on the “Finish Quiz” button to view your score and results. You can also click on the “View Questions” button to review your answers and read the rationales.

Hi, thanks for providing so much useful information and all the test banks!

In RN practice Quiz 1:75 questions, for question 70, wonder if the correct phrase would be “would you NOT expect”?

Hi Cecilia, thanks for letting us know. We have corrected the question. Thank you! 🥰

Thank you for allowing me to practice the nurseslabs questions. The rationales are well explained. Thank you!

Thank you Ruth. Hope you’ve learned a lot from our practice questions. 🥰

God bless you

My name is Samuel. I just started with the RN practice questions. Thank you for the opportunity.

Hi, thank you so much for your generosity. Using this to prepare for my NCLEX and find it very informative. I will keep you posted on how it goes on the D-day

Frequently throughout the quiz, ARDS is given as “Adult Respiratory Distress Syndrome” instead of “Acute Respiratory Distress Syndrome”. This needs to be corrected.

Hi Bella, thanks for bringing this up.

Technically both terms are correct since “adult respiratory distress syndrome” is an earlier name referred to “acute respiratory distress syndrome” to differentiate it from respiratory distress syndrome in neonates and children. For the sake of uniformity, we have edited them to “acute respiratory distress syndrome”. You can read more about it here: https://doi.org/10.1164/ajrccm.149.3.7509706

This is so kind a gesture, remain blessed .The information is very understandable and helpful

I just wanted to thank you for taking the time to generate this website, this is really helpful and is free. God bless you

Thank you for your support and guide toward my nclex preparation.

Thanks for your generosity. I hope to pass my NCCLEX just on the first attempt Grateful for all your help

thanks for the help

Hi Sir, kindly explain why this question – The client with hyperemesis gravidarum is at risk for developing: Metabolic Acidosis is the answer. Thank you.

Quite familiar with the content. Thanks for this selfless initiative.

It has been a nice experience.

I love these practical questions. So helpful.

Thanks very much this is a good help for all students

Thank you for mett this free test which i have passed yesterday afternoon so ,it is greatest pleasure which will help me

Thank you so much for this great initiative.

Thanks for being kind God bless you

thank you for your help and kindness. It’s difficult to go back to school after 22 years of practice as a RN to take the test where you must cover a vast knowledge, which you do not use in everyday working (depends on the special field you are working. so I really appreciate your effort and dedication.

Matt Vera and all those helping us through this free medium, May God richly bless you abundantly. And may all your good desires come through

Hello Vera, I would start by thanking you for the great job you’ve done by helping us in our preparation for the exams. secondly would wish to know wther you have audio clips for lung, heart and abdominal sounds. thank you in advance and if any kindly let mh know pls.

What should we be scoring on these practice questions to let us know that we’re ready for the NCLEX

Do you have LPN test also?

Hi, please visit NCLEX-PN Practice Questions & Test Bank (200 Questions)

Thank you so much Nurse Matt you just don’t know how nurseslabs helped me in my nursing career and i even recommended nurseslabs to some of my friends. God bless you😊

Hi Matt, Thank you for this, indeed very helpful practice questions. Just a query though it says here to limit wrong answers to 15 or less? So is that the passing mark ? about 75-80%

Yes, that is correct. :)

Thank you so much for the practice questions. I learned a lot and it helped me to understand why I was getting my answers wrong. I used Kaplan and Saunders but it wasn’t helping me. I feel more confident than ever to retake my boards.

thank you…. wish this help a lot to my daughter to passed please pray for her God bless!!!

hi Matt, can l still use these question banks to study for the new NGN questions

Hi nurse Matt, May I gave all the questions please, need to prepare study for exam after been off school over 30 years, realize it for not giving up not try again, thank you 🙏

Thank you so much Nurselabs! I passed my NCLEX-RN exam using only your practice 75 question per day questions bank. I have a tight budget and not able to purchased online review and I just focused in answering your practice questions in just 2 weeks!!! Thank you for helping nurses! God Bless you more!

I want to practices questions

Hello Thanks so much for your good heart and good wish for us to practice and pass the NCLEX. Well appreciated. Please after your update I can’t get the answers and rationale to the 75 questions. This will help me to know my ability and where am lagging behind. Please how can this be rectified Thanks Hope

Hi Hope, did you finished all of the questions for the quiz? Your score together with the correct answers and rationale will only show after you’ve completed the whole quiz.

Hello Vera I finished all 75 questions submitted but no answer or rationale came up. I tried upto 3 times same. Even my friend I shared your content with complained of same thing, answers not coming. Please what can we do to access the answers and rationale Thanks Hope

What particular quiz?

Hello, great resource! I am a nurse that wants to help student nurses reach their full potential as nurses and pass the nclex. I would like to utilize your tests and quizzes to help my students learn to critically think through the questions and learn how to put their nursing knowledge to use. Would you allow me to utilize your questions in my content?

Thank you for the kind words! It’s awesome to hear that you’re dedicated to helping student nurses thrive. 🌟

Absolutely, you’re more than welcome to utilize our tests and quizzes for your students! We believe in collaborative efforts to strengthen the nursing community. Just ensure to give appropriate credit when using the content.

Out of curiosity, are there specific areas or topics you find students struggle with the most when preparing for the NCLEX? Maybe we can work together to create even more targeted resources!

Best of luck with your mission to guide the next generation of nurses!

Hi, I am studying for med surg 1 and med surg 2 upcoming exam. Need to pass both exams.

Sounds like you’ve got a busy study schedule ahead! MedSurg can be challenging, but with the right resources and dedication, you’ve got this. If you need specific materials, tips, or just a morale boost, don’t hesitate to reach out. Else, you can do practice all MedSurg exams on our Nursing Test Banks . Rooting for you to ace both exams! 💪

thank you so much… a big help

Hi Glacier and what a nice name!

You’re welcome! Really glad to hear the NCLEX practice questions were a big help. If you’ve got any other areas you’re prepping for or need more practice with, just let me know. Here to support your journey!

Thanks a lot @nurseslabs for this initiative I really believe by the time I sit my exam I will be a great nurse than ever before. I am learning a lot from the questions and explanation.

Hey Charles, You’re very welcome! I’m thrilled to hear that you’re finding our initiative helpful in your journey to becoming a great nurse. Remember, continuous learning is a key part of nursing practice, and it’s fantastic to see your dedication to it. If you ever come across any challenging topics or have questions about specific nursing concepts, please don’t hesitate to reach out. We’re here to support you every step of the way. Keep up the excellent work, and you’ll do great on your exam! 📖🏥🌟

Much help for beginners like us

Thank you so much Nurseslabs, I made it!!! Your practice exams, especially the rationales helped me a LOT…thank you!!!

Wow, that’s awesome news! 🌟 Huge congrats on making it through – your hard work and dedication really paid off. It’s super cool to hear how our practice exams and rationales played a part in getting you across the finish line.

Remember, this is just the beginning of an amazing journey in nursing. The field’s always changing and growing, and so will you. Keep that enthusiasm and curiosity going strong!

If you ever need a hand or just want to share your journey as you dive into your nursing career, give us a shout. We’re always here to cheer you on and lend support.

Big congrats again and best of luck out there!

Thanks for the information

thanks for the guide

Thank you for your kindness.

Thank you very much!

Thanks a lot for this whole range of knowledge; it’s now upon me to work extra hard to be a better nurse outside there or help other nurses be better at service delivery.

Hi Evans, You’re so welcome! I’m glad to hear that the range of knowledge from the NCLEX practice questions is empowering you to aim for excellence in nursing. Remember, every bit of effort counts towards becoming not just a better nurse but also a great mentor to others in the field. If you need more resources or advice along the way, I’m here to help. Keep up the fantastic work!

Please how can I practice questions please 🙏

In practice exam 5 question 22 asks about hyperemesis…. wouldn’t that cause alkalosis (rather than acidosis), since its acid from the stomach that is leaving the body?

Can I. Have free practice questions? NGN questions?

Thank you for the opportunity to know about all the information posted here. I strongly believe I will pass the NCLEX-RN exam at my first attempt through these useful resources available here. Thanks

Thanks a lot Nurselabs for the free test banks! It’s all what I focused to practice and read every rationale in each questions given. I just recently took my NCLEX-RN exam last week and i was able to make it. Your website is what i embraced and felt much comfortable and trusted what is in there. Thanks for helping lots of NURSES! Highly recommended!

Hello Brenda, Congratulations on passing your NCLEX-RN exam! 🎉 That’s a fantastic achievement, and I’m so thrilled to hear that our test banks and rationales were a valuable part of your preparation. Thank you for trusting Nurseslabs and for your kind words—it means a lot to us.

If you have any tips for future test-takers or any other resources you found particularly helpful, we’d love to hear about them. Your feedback not only celebrates your success but also helps us continue to improve and assist other nursing students.

Thanks again for recommending us, and best of luck in your exciting nursing career!

Leave a Comment Cancel reply

  • Nursing Career Guide
  • Premium Login
  • Pass the first time, guaranteed
  • Help Center
  • Back to  all tests
  • Font size Click here to set to default
  • 0   Correct
  • 0   Incorrect
  • 0   Skipped

Next Generation NCLEX Practice Test: Case Studies 3-4

  • 2,500+ NCLEX-Like Questions (including the ones most people fail)
  • 80 Realistic Practice Tests & Marathons
  • 300+ SATA Questions
  • 300+ NCLEX Flashcards
  • Unlimited NCLEX Simulations

NCLEX-RN Fact Sheet

Nursing career guide: how to become a nurse.

Expore resources about studying for your NCLEX and becoming a nurse.

Get the NCLEX Genie App

Download our free iOS or Android app to prepare for your NCLEX exam offline or on the go.

More NCLEX-RN Practice Tests

  • Basic Care and Comfort 1
  • Basic Care and Comfort 2
  • Basic Care & Comfort Marathon
  • Pharmacological & Parenteral Therapies 1
  • Pharmacological & Parenteral Therapies 2
  • Pharmacological & Parenteral Therapies 3
  • Pharmacological & Parenteral Therapies 4
  • Drug Dosage Calculation
  • Pharmacological & Parenteral Marathon
  • Reduction of Risk Potential 1
  • Reduction of Risk Potential 2
  • Reduction of Risk Potential 3
  • Reduction of Risk Potential 4
  • Reduction of Risk Potential Marathon
  • Physiological Adaptation 1
  • Physiological Adaptation 2
  • Physiological Adaptation 3
  • Physiological Adaptation 4
  • Physiological Adaptation Marathon
  • Psychosocial Integrity Test 1
  • Psychosocial Integrity Test 2
  • Psychosocial Integrity Test 3
  • Psychosocial Integrity Test 4
  • Psychosocial Integrity Test 5
  • Psychosocial Marathon
  • Health Promotion and Maintenance 1
  • Health Promotion and Maintenance 2
  • Health Promotion and Maintenance 3
  • Health Promotion and Maintenance 4
  • Health Promotion and Maintenance 5
  • Health Promotion Marathon
  • Management of Care 1
  • Management of Care 2
  • Management of Care 3
  • Management of Care Marathon
  • Safety & Infection Control 1
  • Safety & Infection Control 2
  • Safety & Infection Control 3
  • Safety & Infection Control Marathon
  • NCLEX Exam Simulator

case study next generation nclex sample questions

Breaking Down IV Fluids: The 4 Most Common Types and Their Uses

case study next generation nclex sample questions

How Much Do Nurses Make?

case study next generation nclex sample questions

How to Pass the NCLEX the First Time

case study next generation nclex sample questions

Next Generation NCLEX (NGN) Example Questions

These free practice questions are the best way to familiarize yourself with the style and feel of the new item types that you will encounter on the next generation nclex., frequently asked questions about next gen nclex, when does the next gen nclex start, how are next gen nclex questions different, is the next gen nclex harder, how should i study for the next gen nclex, how many questions is the next gen nclex, what kind of questions are on the next gen nclex, do you get partial credit on next gen nclex, is the new nclex all multiple-choice.

Collaborative Momentum Consulting

Nursing Education Consultants

Collaborative Momentum Consulting

Writing Next Generation NCLEX-Style Case Study Questions

By Susan Sportsman, PhD, RN, ANEF, FAAN

Recently, our blog featured tips for developing Next Generation NCLEX (NGN)-style questions, particularly the revision of test questions faculty already include on their examinations. Revising these questions provides a strategy to adapt current test items to reflect the NGN process, since these knowledge questions represent many questions on the NGN exam.

Now let’s turn our attention to writing NGN Case Study questions . Although only three case studies will be on the NGN examination, all test-takers must respond to the six questions embedded in these case studies, regardless of how many total questions they receive. Case Studies are complex and require some thinking to create. However, I think they are fun to write—the exercise is very much like writing (a truly short) short story! Here are some suggestions for getting started.

  • Begin with the objective(s) that emphasize the content you want students to apply. Keep in mind that the point of case studies (and NGN questions in general) is to evaluate how well students can apply content to a specific clinical situation.
  • Develop a clinical problem that students must address that represents the objective(s) in your course. Ideas for this this problem might come from:
  • Clinical situations highlighted in past tests (or in test banks to which you have access).
  • Your own clinical experiences-or those of your colleagues.
  • Content outlined in the NCLEX Test Design.
  • Case Studies you use for class discussion or assignments for students.
  • NCSBN’s Top Priority Knowledge Statements or Skills
  • The introduction to the case study scenario should include:
  • Client demographics (age, gender, brief and relevant social history)
  • Client history
  • Relevant environmental factors, such as where the client lives (own home, LTC facility, etc.)
  • Normal/abnormal findings (e.g., Physical assessment findings, V/S, Lab values, psychological behaviors)
  • Time considerations for the nurse/client
  • Client/Nurse Interaction
  • Recognize Cues: What matters most?
  • Analyze Cues: What could it mean?
  • Prioritize Hypothesis: Where do I go to start
  • Generate Solution: What can I do?
  • Take Action: What will I do?
  • Evaluate Outcomes: Did it help?

Remember that the case study should represent situations that are appropriate for the level of student you are testing.  Later in the curriculum the questions should represent clinical judgment that a novice nurse is expected to use in actual practice.

Starting the Process of Writing a Case Study

As you begin to write your own case studies, an example might be helpful in integrating the NGN components into an actual case study. The case study below was written for students in an early nursing clinical course, such as Fundamental in Nursing. In addition, the NGN Resources website also provides examples of case studies written at the level of a novice nurse.

Note: The templates we developed for this post can be found on our Resources page . Feel free to use them.

case study next generation nclex sample questions

#1. Recognize Cues (Drag and Drop)

Drag & Drop the 4 cues the RN observed the initial assessment which require following up.

case study next generation nclex sample questions

#2. Analyze Cues (Select All that Apply-N)

The RN is concerned about the client’s confusion. Identify factors that may be causes for this confusion. a. Hospital Environment  * b. Early indication of Alzheimer’s Disease c. Use of Morphine for control of Pain  * d. Anesthesia  * e. Reduction in O2 f. Limited Sleep

case study next generation nclex sample questions

#3. Prioritize Hypothesis (CLOZE)

Complete the following sentence by choosing from the following lists of options:

case study next generation nclex sample questions

#4. Generate Solutions (Matrix)

Confusion continues to be a problem for the client on day three post-surgery.

Which of the possible interventions to the problem are indicated and which are contraindicated?

case study next generation nclex sample questions

The RN notifies the surgeon of elevated TPR, drainage at surgical site, and pain and received orders.

#5. Take Action (Highlighting)

Upon receiving the following physician orders, highlight those which the nurse should implement right away.

case study next generation nclex sample questions

#6. Evaluate Outcomes (Matrix)

The nurse has performed the interventions as ordered by the physician for the client. For each assessment finding, click to specify if the finding indicates that the client’s condition has improved, has not changed, or become worse.

case study next generation nclex sample questions

Case study questions are an important part of the NGN emphasis on measuring test-takers’ clinical judgment competence. Hopefully, the example above will give you some ideas for developing your own case study questions. You may notice that these questions are not exactly formatted like those on the NGN website. Currently, most of us do not have the necessary software to completely mimic the NGN questions. But simple templates can come close. Please don’t let the differences in the format of these questions stop you from using case study questions in your testing and classroom teaching.  Remember, the point of the NGN testing process is to assess new graduates’ ability to use good clinical judgment in a variety of clinical situations.  Giving students the opportunity to practice clinical judgment is key to their success on NCLEX and in clinical practice.

Silvestri, L. (2020) Higher-Cognitive Level Test Question:  A Starting Point for Creating Next Generation NCLEX Test Questions. White Paper. Elsevier.

Share this:

Leave a reply cancel reply, discover more from collaborative momentum consulting.

Subscribe now to keep reading and get access to the full archive.

Type your email…

Continue reading

KeithRN

Writing Case-Based Next-Gen NCLEX Questions

case study next generation nclex sample questions

Related Content

  • Defining Clinical Judgment and Why it’s So Important
  • Frameworks to Teach Clinical Judgment
  • Why are Case Studies Important for Developing Clinical Judgment?
  • How to Transform Classroom Learning
  • Strategies to Transform the Clinical to Develop Clinical Judgment

Developing Clinical Judgment in Simulation

  • How to best develop clinical judgment in nursing students (video series)

Keith Rischer – Ph.D., RN, CCRN, CEN

As a nurse with over 35 years of experience who remained in practice as an educator, I’ve witnessed the gap between how nursing is taught and how it is practiced, and I decided to do something about it! Read more…

The Ultimate Solution to Develop Clinical Judgment Skills

KeithRN’s Think Like a Nurse Membership

Access exclusive active learning resources for faculty and students, including KeithRN Case Studies, making it your go-to resource.

Sign up to receive a free KeithRN NextGen Case Study!

Develop clinical judgment skills with this innovative new case study on asthma to prepare your students for nextgen nclex and practice.

" * " indicates required fields

Keith Rischer

Recommended for you.

case study next generation nclex sample questions

It Was the Worst of Times…and Professional Identity Still Mattered

case study next generation nclex sample questions

In April 2023, the NCSBN released a revamped NCLEX to the world—the Next Generation NCLEX (NGN) —compelling nursing students/grads just like you to rethink your studying strategies! (To learn more about the way NGN questions differ from the previous generation exam, check out: What to expect from Next Generation NCLEX .)

So, to help you adapt and prepare, the team here at Brainscape joined forces with NCLEX prep expert Justine Buick ( www.theNCLEXTutor.com ) who has crafted from scratch a collection of Next Gen NCLEX practice questions , which you'll see shortly.

Combined with our digital flashcards for the NCLEX-RN and NCLEX-PN licensing exams, these constitute a powerful resource you can lean on to take your NCLEX prep to the next level!

Brainscape NCLEX flashcards for nursing students

These flashcards have been carefully curated and vetted to offer nurse students a hyper-efficient way to drill the enormous amounts of content they need to learn and remember in order to answer the more complex questions posed by Next Generation NCLEX questions.

  • Get Brainscape’s Nursing Fundamentals flashcards (for nursing school) 
  • Get Brainscape’s certified NCLEX-RN flashcards
  • Get Brainscape’s certified NCLEX-PN flashcards
  • Get Brainscape’s FNP flashcards
  • Use Brainscape's free NCLEX study planners and exam countdown sheets to plot your path towards and remain razor focused on your study goals!

And if you're still in nursing school, check out our flashcards for all the topics you'll be learning: Pediatrics flashcards , Adult Health flashcards , Pharmacology flashcards , Management of Care flashcards , Maternity flashcards , Mental Health flashcards , and Safety & Infection flashcards .

Now, let’s dive into some Next Generation NCLEX practice questions with, first, a brief aside on the difference between standalone and case study questions...

Standalone vs case study NGN questions

The NCLEX poses questions either as “stand-alone” questions or as “case studies”. In the case of the former, they are—as their name suggests—one-off, discrete questions. Once you finish answering them, you move on to the next question, which isn’t at all related. 

Case studies, on the other hand, ask a series of questions based on the same patient/case and so it serves you to bear in mind your previous answers and maintain a degree of cohesiveness as you work through them.

The first Next Generation NCLEX practice question we’ll be tackling, however, is a standalone bowtie. This is one of the more common types of standalone questions on the NCLEX, which very simply gives you a patient scenario and then you have one question that you have to answer.

NGN Practice Question: Standalone Bowtie

The nurse is caring for a 67-year-old client in the cardiac care unit. The nurse documents the following data in the Nurses’ Notes.

Nurses' Notes

1530: Client was admitted 2 days ago for a scheduled coronary artery bypass graft (CABG) x 2 vessels. No complications during surgery and recovered in the ICU overnight. Client transferred out of the ICU this morning to the cardiac care unit. History of coronary artery disease, hypertension, hyperlipidemia, and osteoporosis. Assessment findings are as follows: alert and oriented x 3; denies shortness of breath, sore throat or cough, lungs clear bilaterally; irregular heart rhythm auscultated, denies chest pain, skin is warm to touch, cap refill < 3 seconds, no edema and peripheral pulses 2+ and equal bilaterally; denies nausea/vomiting, bowel sounds active x 4 quadrants, reports passing flatus and last bowel movement was the morning of surgery; indwelling foley catheter was discontinued this morning and has been voiding clear, yellow urine without complications; up to bathroom 2 times with assistance; gait steady. Client is currently sitting at bedside and reporting a "racing heartbeat" and "feeling faint". Vital signs are HR 142 beats/min, BP 116/78, RR 18 breaths/min, pulse oximetry 96% on room air, and temperature 98.4 F (36.9 C).

Complete the diagram by selecting from the choices below to specify which potential condition the client is most likely experiencing, 2 actions the nurse takes to address that condition, and 2 parameters the nurse monitors to assess the client's progress.

case study next generation nclex sample questions

For the answers...

Simply sign up for Brainscape's nursing flashcard collections (below) and we'll send the detailed answer and rationale to your inbox! These flashcards are FREE to sign up for ; and you can study a certain number of them before being asked to subscribe. But for full access to the Brainscape study experience, which will help you prepare for the NCLEX so much more efficiently , sign up for pro .

NGN Practice Question: Case Study

Now, let's try another type of NGN question: the case study, which, if you remember what we previously explained, poses a series of questions based on the same patient/case. So remember to bear in mind your previous answers and maintain a degree of cohesiveness as you work through them!

The nurse is caring for an older adult in the intermediate care unit.

1000: Client admitted with an exacerbation of chronic obstructive pulmonary disease (COPD). Client has used home oxygen at 3 L/minute by nasal cannula for 4 years but reports increased dyspnea over the last several days. Client increased portable oxygen to 5 L/minute. Currently, the client is fatigued but alert and oriented x 3. Client reports shortness of breath at rest, increased coughing and large amounts of thick, tan sputum. Client has labored breathing and is in the tripod position. Lung sounds with wheezes bilaterally. S1 and S2 heart sounds heard, no murmur or gallop noted. Vital signs obtained. Healthcare provider in to see client and admission laboratory tests completed. Respiratory therapist was in to obtain ABGs.

V ital Signs

case study next generation nclex sample questions

Lab Results

case study next generation nclex sample questions

The nurse assesses the client, obtains the vital signs and reviews the laboratory results. 1. Which laboratory findings require follow-up? Select all that apply.

case study next generation nclex sample questions

2. The nurse reviews the assessment findings and laboratory results. For each client finding, click to select the finding that is most consistent with the condition of hypercapnia or hypoxemia. Each finding may support more than one condition.

case study next generation nclex sample questions

3. Complete the following sentence by choosing from the list of options.The client is at highest risk for developing [Option 1] as evidenced by the client's [Option 2] .

case study next generation nclex sample questions

4. For each finding below, choose the potential nursing intervention that is appropriate for the nurse to include in the plan of care. Each finding may support more than one intervention.

case study next generation nclex sample questions

The nurse reviews the healthcare provider's orders.

  • Continuous pulse oximetry monitoring
  • Oxygen via venturi face mask, titrate to maintain pulse ox >88% • collect sputum culture for culture and sensitivity
  • Repeat ABGs at 1800 today
  • X-ray of chest today
  • albuterol 2.5 mg via inhaler three times a day
  • fluticasone 88 mcg via inhaler three a day guaifenesin 400 mg PO every 4 hours

5. What teaching points does the nurse provide regarding the inhaled medications? Select 3 teaching points.

case study next generation nclex sample questions

The nurse assesses the client after implementing the healthcare provider's orders. 

6. Check whether the assessment findings indicate that the client’s condition is either Improved/Stable or Not Improved .

case study next generation nclex sample questions

NGN Case Study Answers

For the full answers and rationale, include video explanations, simply sign up for Brainscape’s Nursing Fundamentals flashcards (for nursing school) or our certified NCLEX-RN or NCLEX-PN flashcards , which offer you a hyper-efficient way to drill the enormous amounts of content you need to learn in order to answer the more complex questions posed by Next Generation NCLEX questions. (And YES, they’re fully Next Generation NCLEX aligned.)

Once you've signed up, we'll email you the answers. (By the way, these flashcards are FREE to sign up for ; and you can study a certain number of them before being asked to subscribe.)

Finally, for truly excellent advice on excelling in nursing school, taking the NCLEX, and beyond, make sure you follow Brainscape Nursing on YouTube and check out our free Academy of articles and nursing guides !

Flashcards for serious learners .

Next Generation NCLEX An Enhanced NCLEX

case study next generation nclex sample questions

The NGN Project

The NGN launched on April 1, 2023 to better measure nursing candidates’ clinical judgment and decision making abilities through the use of innovative item types. The need to measure this stemmed from information gathered during the 2013-2014 NCSBN Strategic Practice Analysis, which showed newly licensed nurses are increasingly expected to make complex decisions while caring for patients.

NGN Development Flowchart

The Development Process

As part of the NGN project, NCSBN conducted several phases of research that are shown in the model. If the evidence during any individual step indicated that potential innovations would not support the rigor and quality of the NCLEX, the project was reexamined at all levels.

Refine Hypotheses > Prioritize Hypotheses > Generation Solutions

Clinical Judgment Measurement Model

Clinical judgment is critical to nursing. NCSBN developed a model to measure clinical judgement that can also be used as a way of thinking and teaching

case study next generation nclex sample questions

Be Prepared for the NGN

The NCLEX now includes next generation features, using real-world case studies to measure a nurse's ability to think more critically and make the right decisions.

NCLEX Study Guides and Practice Tests

Archives: Next Generation NCLEX (NGN) Questions

Identifying Key Clinical Features of Mononucleosis

Identifying Key Clinical Features of Mononucleosis

Infectious Disease, Mononucleosis, Physical Assessment: NCLEX Practice Quiz

Complications and Management of Tube Feeding

Complications and Management of Tube Feeding

Mastering Tube Feeding Challenges: An NCLEX Practice Explanation

NGN Quiz: Assessment in Gastric Cancer Home Care

NGN Quiz: Assessment in Gastric Cancer Home Care

Gastric Cancer Home Care: Identifying Critical Observations (NCLEX Next Gen Practice Question)

New Year’s Eve 2024 Nursing Prioritization – Next Gen NCLEX Practice Question

New Year’s Eve 2024 Nursing Prioritization – Next Gen NCLEX Practice Question

NCLEX Next Gen Quiz: Medical Management of Epiglottitis

NCLEX Next Gen Quiz: Medical Management of Epiglottitis

  • 1. Prepare the child for tracheotomy.
  • 2. Prepare to administer epinephrine.
  • 3. Prepare the child for a chest radiograph.
  • 4. Assist the primary health care provider with intubation.

NCLEX NGN Quiz on Side Effects of Chlorpromazine

NCLEX NGN Quiz on Side Effects of Chlorpromazine

Chlorpromazine: What Nurses Need to Know

  • (A) Dry mouth
  • (B) Lip smacking
  • (D) Hand tremors
  • (D) Increased urinary output

Critical Lab Monitoring for a Pneumonia Patient on Gentamicin

Critical Lab Monitoring for a Pneumonia Patient on Gentamicin

NCLEX NGN Quiz on Critical Lab Monitoring for a Pneumonia Patient on Gentamicin Topic: Pharmacological Management in Pneumonia System: Renal and Hematological Systems

  • 1. BUN and creatinine
  • 2. Hemoglobin and hematocrit
  • 3. Sodium and potassium
  • 4. Platelet count and clotting time

NCLEX Pharmacology Quiz: Heparin Sodium Therapeutic Effect Monitoring

NCLEX Pharmacology Quiz: Heparin Sodium Therapeutic Effect Monitoring

  • 1. Hematocrit level
  • 2. Hemoglobin level
  • 3. Prothrombin time (PT)
  • 4. Activated partial thromboplastin time (aPTT)

NCLEX NGN Quiz: New Nurse Injection Preparation

NCLEX NGN Quiz: New Nurse Injection Preparation

NCLEX NGN Quiz: Ampule Injection Preparation/Safety

  • 1. Snaps the neck of the ampule gently towards the body.
  • 2. Uses a filter needle when drawing up the ampule contents.
  • 3. Folds gauze around the ampule neck before snapping open.
  • 4. Avoids touching edges of the ampule when inserting needle.

Next Gen Quiz: Neurological Emergencies in Spinal Cord Injury

Next Gen Quiz: Neurological Emergencies in Spinal Cord Injury

Next gen nclex quiz: neurological emergencies in spinal cord injury.

  • 1. autonomic dysreflexia
  • 2. autonomic crisis
  • 3. autonomic shutdown
  • 4. autonomic failure

NCLEX® Changes 2023 — All You Need to Know About the New NCLEX® Format

Table of contents.

case study next generation nclex sample questions

Dive in and start studying for free!

Online NCLEX Review Course

Free Sample Test

NCLEX/NGN Practice Questions

Free Nursing Cheat Sheets

If you are close to completing a nursing degree, you are probably already practicing a few questions daily. Plus, nursing lecturers love to include “NCLEX ® -style questions” on every quiz. Therefore, the announcement of a new NCLEX ® format (large enough to deserve the name “ Next Gen NCLEX ® ”) may have added a bit of anxiety and uncertainty to your study plan.

So what’s this all about? Let’s look at all the available information we have so far.

First Things First: What Is the NCLEX®?

The NCLEX ® is also known as “boards” or “board exams”. This is an exam that all nursing candidates must pass after graduating from nursing school to be able to legally practice as a Licensed Practical Nurse (LPN/LVN) or a Registered Nurse (RN).

The exam’s format, the topics covered, and the passing standards are determined by the National Council of State Boards of Nursing (NCSBN ® ). Depending on the nursing degree you are pursuing, you will have to take one of two versions of the NCLEX ® : the NCLEX-PN ® or the NCLEX-RN ® .

What is the NCLEX® for?

When you’re approaching the senior year of nursing school, it’s easy to think that the sole purpose of the NCLEX ® is to cause extra stress. 

In reality, the exam is meant to certify that a nurse has all the required competencies (that is, clinical judgment and decision-making skills) needed to practice safety. With each candidate receiving the same type exam format and being given questions from each of the Client Needs Categories, each State Nursing Board can be sure, without question , about its members’ baseline knowledge. 

By enrolling in an accredited nursing program, your State Board of Nursing has already looked into your school’s quality standards. They have evaluated the syllabus, clinical hours, simulation hours and faculty/instructor qualifications.

However, it’s possible to complete an accredited nursing program and still be left with some knowledge gaps. Preparing for the exam with a comprehensive study plan is ideal to ensure a successful pass.

How does the exam currently work?

As of 2022, both versions of the NCLEX ® comprise anywhere between 75 and 145 items selected from a pre-set question bank, which include 15 pre-test items that are not scored. These items are a combination of up to 6 types that a candidate may see on the exam.

The 6 item types are:

  • Multiple choice
  • Multiple select all that apply
  • Fill in the blank
  • Ordered response (drag/drop in order)
  • Chart/exhibit

The exam uses a “ computerized adaptive testing ” formula that progressively selects more challenging questions as the candidate progresses through the exam.

All test takers begin with equally “easy” questions. Then, for each question answered correctly, the “adaptive” formula will select a slightly harder one from one of the four Client Needs Categories. For a successful pass, you’ll have to reach the hardest tier in 145 questions or fewer. You won’t be able to flag questions, skip them, or change the answers to past ones. The pass or fail decision will NOT be known at the time the computer cuts off (which can be from 75 to 145). The candidate will receive mailed notification in 6 weeks, unless an additional fee is paid to receive the information earlier.

These questions are designed to cover four areas that are essential for client care:

2023 NCLEX® Changes—What to Expect from this Major Overhaul

The NCLEX ® performs a practice analysis every three years to ensure the content distribution matches with what is seen in everyday nursing practice. There is a large pool of items and the NCSBN states these are rotated periodically, and contain so many items that candidates will not see a duplicated question even if the exam is repeated. 

From the moment it was announced, the Next Gen NCLEX ® (or NGN) has introduced changes within scoring and some new item types. Its purpose is to ensure candidates are able to perform clinical judgment and decision-making as a new nurse.

We know that the NGN will include five new different types of questions. In addition to the current multiple choice questions, you can now expect to see some of these new types:

  • Multiple response (Select all that apply, select N, multiple response group)
  • Highlight (highlight test, highlight table)
  • Drag-and-drop (drag-and-drop cloze, drag-and-drop rationale)
  • Drop down (drop down cloze, drop down table, drop down rationale)
  • Matrix/grid (matrix/grid multiple response, matrix/grid multiple choice)

Finally, the look of the exam will also change. Rather than presenting a short scenario and question for each item, the exam will contain a minimum of three “case studies”. A case study consists of 6 questions pertaining specifically to that case/scenario. If the candidate continues taking questions after the minimum of 75, the candidate may see up to 7 more of the new item types. 

Each of the new type items will be shown on a split-screen , which will show the client information on the left side of the screen and the question on the right side. The case study will imitate real-life medical records, with separate tabs to view (such as health history, lab results, or nursing notes). 

In this way, the relevant and irrelevant information will be presented with each question —similar to real life practice.

Why Is the NCLEX® Changing?

Clearly, a lot of work is being put into redoing the NCLEX ® . So why is the NCSBN ® trying to go beyond their routine reviews?

The need for a new method of assessment clinical judgment was identified in 2017, following a massive nationwide RN Nursing Knowledge Survey . During this survey, it became evident that a nurse’s scope of practice is now much broader. In addition, clients have more acute conditions and caring for them involves more profound knowledge and complex decision-making skills.

The new exam structure and question formats are meant to better “assess the clinical judgment” of all candidates.

Before the Next Gen NCLEX begins in April 2023, a “Special Research Section” has been a volunteer section of the NCLEX for several years, as data is gathered on the new type of questioning. The additional research questions are separate from the NCLEX and are not part of the scoring. This helps to ensure that the new question formats are understandable, appropriately coded, and representative of the latest standard of care.

Related videos

Faqs on the next gen nclex®.

Still have questions? So do most of us! Here is a quick rundown of the most pressing queries we all have about the Next Gen NCLEX®.

Nclex rn ngn qbank

Get Ready for the NCLEX in Time

with Lecturio’s Next Gen NCLEX Preparation Package

When does the Next Gen NCLEX ® start?

The new exams go live on April 1, 2023. 

How does the Next Gen NCLEX ® score the new types of questions?

The exams will continue using the computerized adaptive testing formula, so the total number of questions will vary from person to person. There will be partial scoring applied per question and type. More information on scoring can be found in this publication .

What will the Next Gen NCLEX ® passing grade be?

As in nursing school, there is not an “passing grade”, but the computer will decide on a pass or fail by one of the 3 rules :

  • 95% Confidence Interval Rule
  • Maximum-Length Exam Rule
  • Run-out-of-time (R.O.O.T.) Rule

These rules apply to both the RN and LPN versions of the exams.

Is the new NCLEX® harder?

When it comes to tests, “hard” is very subjective. Because of its emphasis on critical thinking, the NGN will require you to understand disease processes and think like an independent nurse. If you always struggled with memorizing “dry info” but are excellent in case studies, it may be a plus for you!

Still, the new Next Gen NCLEX ® format will require you to start studying differently.

Ultimately, the process of becoming a nurse goes beyond exam day. If you stay on top of your studies throughout your career and jump into your clinicals ready to take the initiative, you will maximize your chances of success. If you feel graduation is looming near, then consider this step-by-step review, that provides illustrated explanations by Prof. Rhonda Lawes in her Med Surg Nursing & Pathophysiology course.  We are here to help you succeed and achieve your dreams!

Medical surgical nursing and pathophysiology

Take the Course: Pathophysiology

Learn all you need to know to provide advanced patient care.

Nclex pharmacology review

NCLEX® Pharmacology Review

This review will prepare you to crush Pharm on the NCLEX®!

A portrait of Ximena, she is smiling into the camera

Ximena is a copywriter and medical interpreter turned Nursing student. She feels very strongly about patient education and about opening healthcare access among women from immigrant and rural communities. During her downtime, she enjoys jogging, RPG gaming, and reading about classical history.

Don't go alone.

Take Lecturio’s support with you and pass the NCLEX with confidence.

Study success starts with the right resources! Lecturio will help you prepare for the NCLEX reliably and efficiently.

Lecturio's Anatomy Course with Prof. Pickering

Further Reading

All about the NCLEX Changes

How to Study for the NCLEX® – and Pass! NGN Tips & Strategies

Nclex® registration guide: how to, when, and costs.

Study with Lecturio for

Medical School

Nursing School

  • Data Privacy
  • Terms and Conditions
  • Legal Information

USMLE™ is a joint program of the Federation of State Medical Boards (FSMB®) and National Board of Medical Examiners (NBME®). MCAT is a registered trademark of the Association of American Medical Colleges (AAMC). NCLEX®, NCLEX-RN®, and NCLEX-PN® are registered trademarks of the National Council of State Boards of Nursing, Inc (NCSBN®). None of the trademark holders are endorsed by nor affiliated with Lecturio.

User Reviews

Get premium to test your knowledge.

Lecturio Premium gives you full access to all content & features

Get Premium to watch all videos

Verify your email now to get a free trial.

Create a free account to test your knowledge

Lecturio Premium gives you full access to all contents and features—including Lecturio’s Qbank with up-to-date board-style questions.

NGN RN NCLEX Exam Mastery 2024 12+

Next generation mock test prep, uladzimir pyzhyk, designed for iphone.

  • Offers In-App Purchases

iPhone Screenshots

Description.

Download to prepare for the certification test. App is specifically designed to provide comprehensive preparation, offering a wealth of resources and practice quizzes. Welcome to NGN RN NCLEX Exam Mastery 2024, the ultimate app to help nurses ace their NGN RN (NEXT GENERATION NCLEX Registered Nurse) exam! Designed specifically for nurses preparing for the RN exam, this comprehensive quiz and prep app offers a user-friendly interface and a wealth of study materials to help you succeed. With NGN RN NCLEX Exam Mastery 2024, you'll have access to a vast question bank covering all key areas of the NGN RN exam. Test your knowledge and practice your skills with interactive quizzes, and track your progress with detailed performance reports. Key Features: 1. Extensive Question Bank: Access a wide range of practice questions covering all areas of the RN exam. 2. Multiple Quiz modes: Take quizzes based on specific topics, difficulty levels, or question types to focus your study efforts and reinforce your weak areas. 3. Performance Tracking: Get detailed performance reports that provide insights into your strengths and weaknesses, so you can optimize your study plan and monitor your progress. 4. Study Materials: Access comprehensive study materials, including explanations, references, and rationales for each question, to deepen your understanding of the exam content. 5. Timed Quiz and Mock Exam: Simulate the real NGN RN exam experience with timed practice exams that help you build your test-taking skills and boost your confidence. Why Choose NGN RN NCLEX Exam Mastery 2024? Comprehensive Exam Coverage: Our app covers all key areas of the RN exam, providing you with a well-rounded preparation to increase your chances of success. Customized Learning: Create personalized quizzes and focus on specific areas that you need to improve, helping you optimize your study efforts and save time. Detailed Performance Reports: Get insights into your strengths and weaknesses through detailed performance reports, allowing you to identify areas that require further review and track your progress. Study Materials: Access comprehensive study materials, including explanations and references, to enhance your understanding of the exam content and reinforce your learning. User-Friendly Interface: Our app features a user-friendly interface that makes studying enjoyable and convenient, allowing you to learn at your own pace and on your own schedule. NGN RN NCLEX Exam Mastery 2024 is a must-have app for nurses who are preparing for the NGN RN exam. Whether you're a seasoned nurse looking to advance your career or a recent graduate entering the field, our app is your comprehensive study companion to help you ace the RN exam and achieve your professional goals. Please note: This app is not affiliated with the official RN exam, and the questions are not taken from the official exam. However, the app is designed to help nurses reinforce their knowledge and test their understanding of the exam content in a convenient and accessible way. Get NGN RN NCLEX Exam Mastery 2024 today and be well-prepared for your NGN-RN exam! You can find more information about our Products and Premium subscription here: Terms of Use - https://examprep.help/terms.html Privacy Policy - https://examprep.help/policy.html

App Privacy

The developer, Uladzimir Pyzhyk , indicated that the app’s privacy practices may include handling of data as described below. For more information, see the developer’s privacy policy .

Data Not Linked to You

The following data may be collected but it is not linked to your identity:

  • Diagnostics

Privacy practices may vary based on, for example, the features you use or your age. Learn More

Information

  • NGN RN Weekly Full Access USD 9.99
  • NGN RN Monthly Full Access USD 22.99
  • Developer Website
  • App Support
  • Privacy Policy

More By This Developer

CNE Nurse Educator Prep 2024

CCM Case Manager Exam Prep Pro

NCLEX RN Mastery Prep 2024

Kansas Broker Exam Prep 2024

AFAA CGFI Exam Test Prep 2024

CM ICPM Manager Exam Prep 2024

You Might Also Like

NCLEX100: Flashcards for nclex

NREMT & NCLEX Mastery

NCLEX RN Questions Tests 2024

NCLEX-RN Genics

COMMENTS

  1. Next Generation NCLEX Case Study Sample Questions

    NGN Case Study Sample Questions and Answers. First, let's take a look at our case study summary below: Case Study Summary: A 68-year-old male is admitted with shortness of breath. He reports difficulty breathing with activity, lying down, or while sleeping. He states that in order to "breathe easier," he has had to sleep in a recliner for ...

  2. NextGen NCLEX Test Bank

    The test bank is composed of case studies with six questions each that follow the NCSBN Clinical Judgment Measurement Model steps: recognize cues. analyze cues. prioritize hypotheses. generate solutions. take action. evaluate outcomes. In addition, seven questions for reviewing bow-tie or trend items are included.

  3. NCLEX Practice Questions: #1 Free NCLEX Test Bank 2024

    The best and free NCLEX practice questions resource to help you achieve success on your NCLEX-RN exam! This guide is updated for 2024! ... Unfolding Case Study. ... Next Generation NCLEX®-style practice questions of all types are illustrated through stand-alone case studies and unfolding case studies. NCSBN Clinical Judgment Measurement Model ...

  4. Practice with NCLEX® & NGN Sample Questions

    Practicing with high-quality Next Generation and NCLEX-style sample questions is the most effective way to prepare for the exam, because if practice feels like the actual exam, then the real thing will feel like practice. ... but should be administered via IV route to maintain NPO status in case of emergency surgery. However, circulation takes ...

  5. Next Generation NCLEX: Case Studies 5-6

    A new Next-Generation NCLEX (NGN) practice test to help NCLEX applicants prepare for the upcoming changes to the exam. Case studies 5-6. ... Each case study consists of a clinical scenario and six questions requiring candidates to make multiple decisions. The items are presented in sequential order, so candidates progress through all six steps ...

  6. Next Generation NCLEX® Practice Questions: Case Studies

    The Next Generation NCLEX® (NGN) includes new unfolding case studies. During this brief video we will walk-through a NGN-style case study and share insights ...

  7. Next Generation NCLEX Practice Test: Case Studies 3-4

    Our practice test follows the NGN's minimum and maximum length of the exam, which is 85 to 150 questions, with 70 to 135 scored and 15 unscored questions. Candidates can take up to five hours to complete the exam, with the NGN case studies being delivered according to the computerized adaptive test (CAT) model.

  8. Tips for Answering Next Gen NCLEX® Case Studies

    Practice Next Gen NCLEX Case Study Questions. NGN case studies take some time to get used to. At first, half the battle is understanding the item types and how to identify relevant information. But, as the saying goes, practice makes perfect! By practicing NGN case studies before your exam, you can get familiar with the testing layout, develop ...

  9. Next Gen NCLEX Practice Questions

    Next Generation NCLEX (NGN) Example Questions. These free practice questions are the best way to familiarize yourself with the style and feel of the new item types that you will encounter on the Next Generation NCLEX. Try NGN Sample Questions.

  10. Free Next Gen NCLEX® Practice Questions

    Free Next Generation NCLEX-RN Sample Test. Lecturio's Free Next Generation NCLEX-RN Sample Test was created to help students measure their preparedness and get firsthand experience in a simulated NGN testing. This sample test comprises 30 realistic questions that blend new NGN item types with traditional NCLEX-RN questions.

  11. Writing Next Generation NCLEX-Style Case Study Questions

    Client/Nurse Interaction. The six questions in the case study should focus on the application of one of the cognitive skills highlighted in the Clinical Judgment Measurement Model (CJMM). Silvestri (2020) identifies questions that test-writers (and test-takers) ask themselves to understand each of the six cognitive skills.

  12. Key Strategies to Answer Next Gen NCLEX Case Study Questions

    For both the Registered Nurse (RN) and Practical Nurse (PN) exam, the Next Gen NCLEX takes up to 5 hours to complete and has up to 150 questions. Test takers must answer at least 85 questions, with 70 scored and 15 unscored. If the test taker answers all 150 questions, 135 of those are scored, and 15 are unscored.

  13. Writing Case-Based Next-Gen NCLEX Questions

    You can create your own case-based scenarios and clinical reasoning questions to help your students develop clinical judgment, preparing them for both practice and NGN. Students learn best when learning from salient, case studies which communicate a story, similar to what they will experience in practice. When writing scenarios for tests, it ...

  14. Next Generation NCLEX (NGN) Sample Questions Case Study Practice

    Next Generation NCLEX (NGN) sample questions case study practice review. In this video, you'll see some sample questions for NCLEX practice, similar to the c...

  15. Can you answer these free NGN practice questions for the NCLEX?

    Now, let's dive into some Next Generation NCLEX practice questions with, first, a brief aside on the difference between standalone and case study questions... Standalone vs case study NGN questions. The NCLEX poses questions either as "stand-alone" questions or as "case studies". In the case of the former, they are—as their name ...

  16. Next Generation NCLEX (NGN)

    The NCLEX now includes next generation features, using real-world case studies to measure a nurse's ability to think more critically and make the right decisions. The NCLEX is the world's premier licensure exam and uses computerized adaptive testing (CAT) technology to deliver the exam, ensuring a valid and reliable measurement of nursing ...

  17. Next Gen NCLEX® (NGN): Free Nursing Educator Resources

    Free NGN Practice Questions. Experience the new item types on the Next Gen NCLEX with our newly-released practice questions. Below, you can read and watch webinars about each new item type on the NGN. ... How to Write a Next Generation NCLEX® Case Study. The new question types on the Next Gen NCLEX® will include case study information. The ...

  18. Next Generation NCLEX (NGN) Practice Questions

    Prepare for the Next Generation NCLEX (NGN) with our comprehensive question bank. Our questions are designed to test your critical thinking skills and to prepare you for the real exam. We offer a variety of question types, including multiple choice, fill-in-the-blank, and drag-and-drop. We also offer explanations for all of our questions, so you can learn from your mistakes.

  19. Next Generation NCLEX

    The NCSBN has introduced a new format for the NCLEX® exam is now available. The Next Generation NCLEX exam includes a new case study-based format and new question types. The NGN Explainer videos offer a great overview, with visual representations of all the questions and item types! View all NGN item types. Contact Sales Representative

  20. NCLEX Changes 2023

    If you are close to completing a nursing degree, you are probably already practicing a few questions daily. Plus, nursing lecturers love to include "NCLEX ®-style questions" on every quiz.Therefore, the announcement of a new NCLEX ® format (large enough to deserve the name "Next Gen NCLEX ® ") may have added a bit of anxiety and uncertainty to your study plan.

  21. Next Gen NCLEX Case Study Item Type

    12 NGN case study item types. The most significant change to the test is the introduction of 12 new case study item types and 2 stand-alone item types. These include: Matrix multiple-choice. Matrix multiple-response. Multiple-response: Select all that apply. Multiple-response: Select N. Multiple-response: Grouping.

  22. Next Gen NCLEX Question Types: Stand-alone vs. Case Study

    The Next-gen NCLEX (NGN) will be implementing the new format for test-takers starting in April of 2023. This module looks at stand-alone and case study questions, techniques on how to answer these ...

  23. NGN RN NCLEX Exam Mastery 2024 12+

    Welcome to NGN RN NCLEX Exam Mastery 2024, the ultimate app to help nurses ace their NGN RN (NEXT GENERATION NCLEX Registered Nurse) exam! Designed specifically for nurses preparing for the RN exam, this comprehensive quiz and prep app offers a user-friendly interface and a wealth of study materials to help you succeed.